The Stable Matching Lattice under Changed Preferences, and Associated Algorithms

Autor: Gangam, Rohith Reddy, Mai, Tung, Raju, Nitya, Vazirani, Vijay V.
Rok vydání: 2023
Předmět:
Druh dokumentu: Working Paper
Popis: [MV18] introduced a fundamental new algorithmic question on stable matching, namely finding a matching that is stable under two ``nearby'' instances, where ``nearby'' meant that in going from instance $A$ to $B$, only one agent changes its preference list. By first establishing a sequence of structural results on the lattices of $A$ and $B$, [MV18] and [GMRV22] settled all algorithmic questions related to this case. The current paper essentially settles the general case. Assume that instance $B$ is obtained from $A$, both on $n$ workers and $n$ firms, via changes in the preferences of $p$ workers and $q$ firms. If so, we will denote the change by $(p, q)$. Thus [MV18] and [GMRV22] settled the case $(0, 1)$, since they adopt the convention that one firm changes its preferences. Let $\mathcal{M}_A$ and $\mathcal{M}_B$ be the sets of stable matchings of instances $A$ and $B$, and let $\mathcal{L}_A$ and $\mathcal{L}_B$ be their lattices. Our results are: 1. For $(0, n)$, $\mathcal{M}_A \cap \mathcal{M}_B$ is a sublattice of $\mathcal{L}_A$ and of $\mathcal{L}_B$. We can efficiently obtain the worker-optimal and firm-optimal stable matchings in $\mathcal{M}_A \cap \mathcal{M}_B$. We also obtain the associated partial order, as promised by Birkhoff's Representation Theorem, and use it to enumerate these matchings with polynomial delay. 2. For $(1, n)$, the only missing results are the partial order and enumeration. 3. We give an example with $(2, 2)$ for which $\mathcal{M}_A \cap \mathcal{M}_B$ fails to be a sublattice of $\mathcal{L}_A$. In light of the fact that for $(n, n)$, determining if $(\mathcal{M}_A \cap \mathcal{M}_B) = \emptyset$ is NP-hard [MO19], a number of open questions arise; in particular, closing the gap between $(2, 2)$ and $(n, n)$.
Comment: 28 pages and 12 figures
Databáze: arXiv